Difference between revisions of "Aufgaben:Exercise 3.3: Noise at Channel Equalization"

From LNTwww
m (Text replacement - "Category:Aufgaben zu Digitalsignalübertragung" to "Category:Digital Signal Transmission: Exercises")
Line 1: Line 1:
  
{{quiz-Header|Buchseite=Digitalsignalübertragung/Ber%C3%BCcksichtigung_von_Kanalverzerrungen_und_Entzerrung
+
{{quiz-Header|Buchseite=Digital_Signal_Transmission/Consideration_of_Channel_Distortion_and_Equalization
 
}}
 
}}
  
[[File:P_ID1407__Dig_A_3_3.png |right|frame|Rausch–LDS vor dem Entscheider]]
+
[[File:P_ID1407__Dig_A_3_3.png |right|frame|Noise PSD before the decision]]
Wir betrachten zwei unterschiedliche Systemvarianten, die beide NRZ–Rechteck–Sendeimpulse benutzen und durch AWGN–Rauschen beeinträchtigt werden.  
+
We consider two different system variants, both of which use NRZ rectangular transmission pulses and are affected by AWGN noise.
*In beiden Fällen wird zur Rauschleistungsbegrenzung ein Gaußtiefpass
+
*In both cases, a Gaussian low-pass filter is used to limit noise power
 
:$$H_{\rm G}(f) = {\rm exp}(- \pi \cdot
 
:$$H_{\rm G}(f) = {\rm exp}(- \pi \cdot
 
\frac{f^2}{(2f_{\rm G})^2})$$
 
\frac{f^2}{(2f_{\rm G})^2})$$
  
:mit der normierten Grenzfrequenz  $f_{\rm G} \cdot T = 0.35$  verwendet, so dass beide Systeme mit  $\ddot{o}(T_{\rm D} = 0) = 0.478 \cdot s_0$  auch die gleiche Augenöffnung aufweisen.
+
:with normalized cutoff frequency  $f_{\rm G} \cdot T = 0.35$  is used, so that both systems also have the same eye opening with  $\ddot{o}(T_{\rm D} = 0) = 0.478 \cdot s_0$.   
*Die pro Bit aufgewendete Sendeenergie  $E_{\rm B} = s_0^2 \cdot T$  ist um den Faktor  $10^9$  größer als die Rauschleistungsdichte  $N_0$   ⇒   $10\cdot {\rm lg} \, E_{\rm B}/N_0 = 90 \, {\rm dB}$.
+
*The transmitted energy  $E_{\rm B} = s_0^2 \cdot T$  spent per bit is larger than the noise power density  $N_0$   ⇒   $10\cdot {\rm lg} \, E_{\rm B}/N_0 = 90 \, {\rm dB}$ by a factor of  $10^9$ .
  
  
Die beiden Systeme unterscheiden sich wie folgt:  
+
The two systems differ as follows:
* Der Kanalfrequenzgang von System  $\rm A$  ist frequenzunabhängig:   $H_{\rm K}(f) = \alpha$. Für das Empfangsfilter ist demnach  $H_{\rm E}(f) = H_{\rm G}(f)/\alpha$  anzusetzen, so dass für die Detektionsrauschleistung gilt:
+
* The channel frequency response of system  $\rm A$  is frequency independent:   $H_{\rm K}(f) = \alpha$. Accordingly,  $H_{\rm E}(f) = H_{\rm G}(f)/\alpha$  must be assumed for the receiver filter, so that the following applies to the detection noise power:
 
:$$\sigma_d^2 = {N_0}/{2} \cdot \int_{-\infty}^{+\infty}
 
:$$\sigma_d^2 = {N_0}/{2} \cdot \int_{-\infty}^{+\infty}
 
|H_{\rm E}(f)|^2 \,{\rm d} f = \frac{N_0 \cdot f_{\rm G}}{\sqrt{2}
 
|H_{\rm E}(f)|^2 \,{\rm d} f = \frac{N_0 \cdot f_{\rm G}}{\sqrt{2}
 
\cdot \alpha^2} \hspace{0.05cm}.$$
 
\cdot \alpha^2} \hspace{0.05cm}.$$
* Dagegen ist für System  $\rm B$  ein Koaxialkabel mit der charakteristischen Dämpfung (bei der halben Bitrate)  $a_* = 80 \, {\rm dB}$  $($bzw.  $9.2 \, {\rm Np})$  vorausgesetzt, so dass für den Betragsfrequenzgang gilt:
+
* In contrast, system  $\rm B$  assumes a coaxial cable with characteristic attenuation (at half the bit rate)  $a_* = 80 \, {\rm dB}$  $($or  $9.2 \, {\rm Np})$  so that the magnitude frequency response is:
 
:$$|H_{\rm K}(f)| = {\rm e}^{- 9.2 \hspace{0.05cm} \cdot
 
:$$|H_{\rm K}(f)| = {\rm e}^{- 9.2 \hspace{0.05cm} \cdot
 
\hspace{0.05cm}\sqrt{2 f T}}\hspace{0.05cm}.$$
 
\hspace{0.05cm}\sqrt{2 f T}}\hspace{0.05cm}.$$
* Somit lautet die Gleichung für die Rauschleistungsdichte vor dem Entscheider $($mit  $f_{\rm G} \cdot T = 0.35)$:
+
* Thus, the equation for the noise power density before the decision $($with  $f_{\rm G} \cdot T = 0.35)$ is:
 
:$${\it \Phi}_{d{\rm N}}(f) = {N_0}/{2} \cdot \frac{|H_{\rm G
 
:$${\it \Phi}_{d{\rm N}}(f) = {N_0}/{2} \cdot \frac{|H_{\rm G
 
}(f)|^2}{|H_{\rm K}(f)|^2} = {N_0}/{2} \cdot {\rm exp}\left
 
}(f)|^2}{|H_{\rm K}(f)|^2} = {N_0}/{2} \cdot {\rm exp}\left
Line 27: Line 27:
 
\right ] \hspace{0.05cm}.$$
 
\right ] \hspace{0.05cm}.$$
  
Dieser Funktionsverlauf  $\rm B$  ist in obiger Grafik rot dargestellt. Die Rauchleistungsdichte für das System  $\rm A$  ist blau gezeichnet.
+
This function curve  $\rm B$  is shown in red in the above graph. The noise power density for system  $\rm A$  is drawn in blue.
  
Für das System  $\rm B$  wurde messtechnisch die ungünstigste Fehlerwahrscheinlichkeit
+
For the system  $\rm B$,  the worst-case error probability
 
:$$p_{\rm U} = {\rm Q} \left( \sqrt{\rho_{\rm U}}
 
:$$p_{\rm U} = {\rm Q} \left( \sqrt{\rho_{\rm U}}
   \right) \hspace{0.2cm}{\rm mit} \hspace{0.2cm} \rho_{\rm U} = \frac{[\ddot{o}(T_{\rm D})/2]^2}{ \sigma_d^2}$$
+
   \right) \hspace{0.2cm}{\rm with} \hspace{0.2cm} \rho_{\rm U} = \frac{[\ddot{o}(T_{\rm D})/2]^2}{ \sigma_d^2}$$
  
bestimmt. Die Messung ergab  $p_{\rm U} = 4 \cdot 10^{\rm -8}$, was dem Störabstand  $10 \cdot {\rm lg} \, \rho_{\rm U} = 14.8 \, {\rm dB}$  entspricht.
+
was determined. The measurement resulted in  $p_{\rm U} = 4 \cdot 10^{\rm -8}$, which corresponds to the signal-to-noise ratio  $10 \cdot {\rm lg} \, \rho_{\rm U} = 14.8 \, {\rm dB}$.   
  
  
Line 40: Line 40:
  
  
''Hinweise:''  
+
''Notes:''  
*Die Aufgabe gehört zum  Kapitel  [[Digitalsignal%C3%BCbertragung/Ber%C3%BCcksichtigung_von_Kanalverzerrungen_und_Entzerrung|Berücksichtigung von Kanalverzerrungen und Entzerrung]].
+
*The exercise belongs to the chapter  [[Digital_Signal_Transmission/Consideration_of_Channel_Distortion_and_Equalization|Consideration of Channel Distortion and Equalization]].
* Verwenden Sie zur numerischen Auswertung der Q–Funktion das Interaktionsmodul  [[Applets:Komplementäre_Gaußsche_Fehlerfunktionen|Komplementäre Gaußsche Fehlerfunktionen]].
+
* Use the  [[Applets:Komplementäre_Gaußsche_Fehlerfunktionen|Complementary Gaussian Error Functions]] interaction module for numerical evaluation of the Q function.
 
   
 
   
  
  
  
===Fragebogen===
+
===Questions===
  
 
<quiz display=simple>
 
<quiz display=simple>
{Welcher (normierter) Störeffektivwert tritt bei System &nbsp;$\rm B$&nbsp; auf?
+
{What (normalized) noise rms value occurs in system &nbsp;$\rm B$?&nbsp;
 
|type="{}"}
 
|type="{}"}
 
$\sigma_d/s_0 \ = \ $ { 0.044 3% }
 
$\sigma_d/s_0 \ = \ $ { 0.044 3% }
  
{Welcher Störeffektivwert tritt bei System &nbsp;$\rm A$&nbsp; auf, wenn dieses zur genau gleichen (ungünstigsten) Fehlerwahrscheinlichkeit wie das System &nbsp;$\rm B$&nbsp; führt?
+
{What noise rms value occurs for system &nbsp;$\rm A$&nbsp; when it leads to exactly the same (worst-case) error probability as system &nbsp;$\rm B$?&nbsp;
 
|type="{}"}
 
|type="{}"}
 
$\sigma_d/s_0 \ = \ $ { 0.044 3% }
 
$\sigma_d/s_0 \ = \ $ { 0.044 3% }
  
{Mit welchem Dämpfungsfaktor &nbsp;$\alpha$&nbsp; ist das System  &nbsp;$\rm A$&nbsp; dem System  &nbsp;$\rm B$&nbsp; bezüglich der (ungünstigsten) Fehlerwahrscheinlichkeit äquivalent?
+
{By what attenuation factor &nbsp;$\alpha$&nbsp; is system &nbsp;$\rm A$&nbsp; equivalent to system &nbsp;$\rm B$&nbsp; in terms of (worst-case) error probability?
 
|type="{}"}
 
|type="{}"}
 
$20 \cdot {\rm lg} \ \alpha \ = \ $ { -70.967--66.833 } ${\ \rm dB}$
 
$20 \cdot {\rm lg} \ \alpha \ = \ $ { -70.967--66.833 } ${\ \rm dB}$
  
{Wie groß ist die auf &nbsp;$N_0/2$&nbsp; bezogene Rauschleistungsdichte &nbsp;$($bei &nbsp;$f = 0)$&nbsp; vor dem Entscheider für System &nbsp;$\rm A$&nbsp; bzw. System &nbsp;$\rm B$?
+
{What is the noise power density referenced to &nbsp;$N_0/2$&nbsp; &nbsp;$($at &nbsp;$f = 0)$&nbsp; before the decision for system &nbsp;$\rm A$&nbsp; and system &nbsp;$\rm B$?
 
|type="{}"}
 
|type="{}"}
 
$\text{System A:}\hspace{0.4cm} {\it \Phi}_{d \rm N} (f = 0)/(N_0/2) \ = \ $ { 7.8 3% } $\ \cdot 10^6$
 
$\text{System A:}\hspace{0.4cm} {\it \Phi}_{d \rm N} (f = 0)/(N_0/2) \ = \ $ { 7.8 3% } $\ \cdot 10^6$
 
$\text{System B:}\hspace{0.42cm} {\it \Phi}_{d \rm N} (f = 0)/(N_0/2)  \ = \ $ { 1 3% } $\ \cdot 10^0$
 
$\text{System B:}\hspace{0.42cm} {\it \Phi}_{d \rm N} (f = 0)/(N_0/2)  \ = \ $ { 1 3% } $\ \cdot 10^0$
  
{Für den Rest der Aufgabe betrachten wir ausschließlich das System &nbsp;$\rm B$. Bei welcher Frequenz &nbsp;$f_{\rm max}$&nbsp; besitzt &nbsp;${\it \Phi}_{d \rm N}(f)$&nbsp; sein Maximum?
+
{For the rest of the exercise, we will only consider system &nbsp;$\rm B$. At what frequency &nbsp;$f_{\rm max}$&nbsp; does &nbsp;${\it \Phi}_{d \rm N}(f)$&nbsp; have its maximum?
 
|type="{}"}
 
|type="{}"}
 
$f_{\rm max} \cdot T\ = \ ${ 0.63 3% }
 
$f_{\rm max} \cdot T\ = \ ${ 0.63 3% }
  
{Um welchen Faktor ist die Rauschleistungsdichte bei der Frequenz &nbsp;$f_{\rm max}$&nbsp; größer als bei &nbsp;$f = 0$?
+
{By what factor is the noise power density at frequency &nbsp;$f_{\rm max}$&nbsp; greater than at &nbsp;$f = 0$?
 
|type="{}"}
 
|type="{}"}
 
${\it \Phi}_{d \rm N}(f_{\rm max})/{\it \Phi}_{d \rm N}(0)\ = \ $ { 5.4 3% } $\ \cdot 10^6$
 
${\it \Phi}_{d \rm N}(f_{\rm max})/{\it \Phi}_{d \rm N}(0)\ = \ $ { 5.4 3% } $\ \cdot 10^6$
 
</quiz>
 
</quiz>
  
===Musterlösung===
+
===Solution===
 
{{ML-Kopf}}
 
{{ML-Kopf}}
'''(1)'''&nbsp; Aus $10 \cdot {\rm lg} \, \rho_{\rm U} = 14.8 \, {\rm dB}$ folgt $\rho_{\rm U} = 10^{\rm 1.48} &asymp; 30.2$ und weiter mit der angegebenen Gleichung:
+
'''(1)'''&nbsp; From $10 \cdot {\rm lg} \, \rho_{\rm U} = 14.8 \, {\rm dB}$ follows $\rho_{\rm U} = 10^{\rm 1.48} &asymp; 30.2$ and continue with the given equation:
 
:$$\sqrt{\rho_{\rm U}} = \frac{\ddot{o}(T_{\rm D})/2}{ \sigma_d}\hspace{0.3cm}\Rightarrow
 
:$$\sqrt{\rho_{\rm U}} = \frac{\ddot{o}(T_{\rm D})/2}{ \sigma_d}\hspace{0.3cm}\Rightarrow
 
\hspace{0.3cm} \sigma_d = \frac{0.478 \cdot s_0/2}{ \sqrt{30.2}}
 
\hspace{0.3cm} \sigma_d = \frac{0.478 \cdot s_0/2}{ \sqrt{30.2}}
Line 84: Line 84:
  
  
'''(2)'''&nbsp; Bei gleicher Fehlerwahrscheinlichkeit $p_{\rm U}$ (und damit gleichem $\rho_{\rm U}$) muss $\sigma_d$ genau den gleichen Wert besitzen wie in der Teilaufgabe '''(1)''' berechnet, da auch die Augenöffnung gleich bleibt &nbsp; &#8658; &nbsp; $\sigma_d/s_0 \underline{= 0.044}.$
+
'''(2)'''&nbsp; With the same error probability $p_{\rm U}$ (and thus the same $\rho_{\rm U}$), $\sigma_d$ must have exactly the same value as calculated in subtask '''(1)''', since the eye opening also remains the same &nbsp; &#8658; &nbsp; $\sigma_d/s_0 \underline{= 0.044}.$
  
  
'''(3)'''&nbsp; Entsprechend dem Angabenblatt gilt:
+
'''(3)'''&nbsp; According to the specification section:
 
:$$\alpha^2  =  \frac{N_0 \cdot f_{\rm G}}{\sqrt{2} \cdot \sigma_d^2}
 
:$$\alpha^2  =  \frac{N_0 \cdot f_{\rm G}}{\sqrt{2} \cdot \sigma_d^2}
 
= \frac{10^{-9} \cdot s_0^2 \cdot T \cdot f_{\rm G}}{\sqrt{2}
 
= \frac{10^{-9} \cdot s_0^2 \cdot T \cdot f_{\rm G}}{\sqrt{2}
Line 96: Line 96:
 
  \hspace{0.05cm}.$$
 
  \hspace{0.05cm}.$$
  
In ${\rm dB}$ ausgedrückt erhält man somit
+
Expressed in ${\rm dB}$, one thus obtains
 
:$$20 \cdot {\rm lg}\hspace{0.1cm}\alpha = 10 \cdot {\rm lg}\hspace{0.1cm}\alpha^2 =
 
:$$20 \cdot {\rm lg}\hspace{0.1cm}\alpha = 10 \cdot {\rm lg}\hspace{0.1cm}\alpha^2 =
 
   -70\,{\rm dB}\hspace{0.1cm}+\hspace{0.1cm}10 \cdot {\rm lg}\hspace{0.1cm}1.28\hspace{0.15cm}\underline { =
 
   -70\,{\rm dB}\hspace{0.1cm}+\hspace{0.1cm}10 \cdot {\rm lg}\hspace{0.1cm}1.28\hspace{0.15cm}\underline { =
Line 102: Line 102:
  
  
'''(4)'''&nbsp; Beim System &nbsp;$\rm B$&nbsp; ist wegen $H_{\rm E}(f = 0) = 1$ der normierte Wert gleich $1$, das heißt, es ist ${\it \Phi}_{d \rm N}(f = 0) = N_0/2$.  
+
'''(4)'''&nbsp; For system &nbsp;$\rm B$,&nbsp; because $H_{\rm E}(f = 0) = 1$, the normalized value is equal to $1$, that means, it is ${\it \Phi}_{d \rm N}(f = 0) = N_0/2$.  
  
Dagegen ist bei System &nbsp;$\rm A$&nbsp; dieser Wert aufgrund der Komponenten der frequenzunabhängigen Kabeldämpfung $\alpha$ um $1/\alpha^2$ größer:
+
In contrast, for system &nbsp;$\rm A$,&nbsp; this value is larger by $1/\alpha^2$ due to the components of the frequency-independent cable attenuation $\alpha$:
 
:$${\rm System}\hspace{0.15cm}{\rm A:}\hspace{0.1cm}\frac{{\it \Phi}_{d{\rm N}}(f = 0)}{N_0/2}  = \frac{1}{\alpha^2} \hspace{0.15cm}\underline {\approx 7.8 \cdot 10^{6}} \hspace{0.05cm}, \hspace{1.05cm}{\rm System\hspace{0.15cm}B}: \frac{{\it \Phi}_{d \rm N}(f = 0)}{N_0/2} \, \underline {= 1}.$$
 
:$${\rm System}\hspace{0.15cm}{\rm A:}\hspace{0.1cm}\frac{{\it \Phi}_{d{\rm N}}(f = 0)}{N_0/2}  = \frac{1}{\alpha^2} \hspace{0.15cm}\underline {\approx 7.8 \cdot 10^{6}} \hspace{0.05cm}, \hspace{1.05cm}{\rm System\hspace{0.15cm}B}: \frac{{\it \Phi}_{d \rm N}(f = 0)}{N_0/2} \, \underline {= 1}.$$
  
  
'''(5)'''&nbsp; ${\it \Phi}_{d \rm N}(f)$ ist maximal, wenn der Exponent
+
'''(5)'''&nbsp; ${\it \Phi}_{d \rm N}(f)$ is maximal if the exponent
 
:$$18.4 \cdot \sqrt{2  f  T} - 2\pi \cdot \frac{(f \cdot T)^2}{0.49}$$
 
:$$18.4 \cdot \sqrt{2  f  T} - 2\pi \cdot \frac{(f \cdot T)^2}{0.49}$$
  
den maximalen Wert besitzt. Mit $x = f \cdot T$ gilt somit für die Optimierungsfunktion:
+
has the maximum value. Thus, with $x = f \cdot T$, the optimization function is:
 
:$$y(x) = 26.022 \cdot  \sqrt{x} - 12.823 \cdot x^2 \approx 26 \cdot
 
:$$y(x) = 26.022 \cdot  \sqrt{x} - 12.823 \cdot x^2 \approx 26 \cdot
 
\sqrt{x} - 13 \cdot x^2 \hspace{0.3cm}
 
\sqrt{x} - 13 \cdot x^2 \hspace{0.3cm}
Line 122: Line 122:
 
\hspace{0.05cm}.$$
 
\hspace{0.05cm}.$$
  
Damit ergibt sich $f_{\rm max} \cdot T\hspace{0.15cm}\underline {\approx 0.63}$.
+
This gives $f_{\rm max} \cdot T\hspace{0.15cm}\underline {\approx 0.63}$.
  
  
'''(6)'''&nbsp; Mit $x_{\rm max} = 0.63$ erhält man den Funktionswert
+
'''(6)'''&nbsp; With $x_{\rm max} = 0.63$ we get the function value
  
 
:$$y(x_{\rm max})  \approx 26 \cdot \sqrt{0.63} - 13 \cdot 0.63^2
 
:$$y(x_{\rm max})  \approx 26 \cdot \sqrt{0.63} - 13 \cdot 0.63^2
 
\hspace{0.15cm}\underline {\approx 15.477}.$$
 
\hspace{0.15cm}\underline {\approx 15.477}.$$
[[File:P_ID1408__Dig_A_3_3f.png|frame|right|Rauschanteil $d_{\rm N}(t)$]]
+
[[File:P_ID1408__Dig_A_3_3f.png|frame|right|Noise component $d_{\rm N}(t)$]]
Daraus folgt:  
+
It follows:
*Die Rauschleistungsdichte ist bei der (normierten) Frequenz $f \cdot T \approx 0.63$ um den Faktor $e^{\rm 15.5} \underline{\approx 5.4 \cdot 10^6}$ größer ist als bei der Frequenz $f = 0$.  
+
*The noise power density at the (normalized) frequency $f \cdot T \approx 0.63$ is larger than at the frequency $e^{\rm 15.5} \underline{\approx 5.4 \cdot 10^6}$ by a factor of $f = 0$.  
  
*Im Rauschanteil $d_{\rm N}(t)$ überwiegen somit periodische Anteile mit der Periodendauer $T_0 \approx 1.6 \cdot T$.  
+
*Thus, periodic components with period $T_0 \approx 1.6 \cdot T$ predominate in the noise component $d_{\rm N}(t)$.  
*Die Grafik zeigt eine Simulation und bestätigt dieses Ergebnis.
+
*The graph shows a simulation and confirms this result.
  
 
{{ML-Fuß}}
 
{{ML-Fuß}}

Revision as of 15:12, 2 May 2022

Noise PSD before the decision

We consider two different system variants, both of which use NRZ rectangular transmission pulses and are affected by AWGN noise.

  • In both cases, a Gaussian low-pass filter is used to limit noise power
$$H_{\rm G}(f) = {\rm exp}(- \pi \cdot \frac{f^2}{(2f_{\rm G})^2})$$
with normalized cutoff frequency  $f_{\rm G} \cdot T = 0.35$  is used, so that both systems also have the same eye opening with  $\ddot{o}(T_{\rm D} = 0) = 0.478 \cdot s_0$. 
  • The transmitted energy  $E_{\rm B} = s_0^2 \cdot T$  spent per bit is larger than the noise power density  $N_0$   ⇒   $10\cdot {\rm lg} \, E_{\rm B}/N_0 = 90 \, {\rm dB}$ by a factor of  $10^9$ .


The two systems differ as follows:

  • The channel frequency response of system  $\rm A$  is frequency independent:   $H_{\rm K}(f) = \alpha$. Accordingly,  $H_{\rm E}(f) = H_{\rm G}(f)/\alpha$  must be assumed for the receiver filter, so that the following applies to the detection noise power:
$$\sigma_d^2 = {N_0}/{2} \cdot \int_{-\infty}^{+\infty} |H_{\rm E}(f)|^2 \,{\rm d} f = \frac{N_0 \cdot f_{\rm G}}{\sqrt{2} \cdot \alpha^2} \hspace{0.05cm}.$$
  • In contrast, system  $\rm B$  assumes a coaxial cable with characteristic attenuation (at half the bit rate)  $a_* = 80 \, {\rm dB}$  $($or  $9.2 \, {\rm Np})$  so that the magnitude frequency response is:
$$|H_{\rm K}(f)| = {\rm e}^{- 9.2 \hspace{0.05cm} \cdot \hspace{0.05cm}\sqrt{2 f T}}\hspace{0.05cm}.$$
  • Thus, the equation for the noise power density before the decision $($with  $f_{\rm G} \cdot T = 0.35)$ is:
$${\it \Phi}_{d{\rm N}}(f) = {N_0}/{2} \cdot \frac{|H_{\rm G }(f)|^2}{|H_{\rm K}(f)|^2} = {N_0}/{2} \cdot {\rm exp}\left [18.4 \cdot \sqrt{2 f T} - 2\pi \cdot \frac{(f \cdot T)^2}{(2 \cdot 0.35)^2} \right ] \hspace{0.05cm}.$$

This function curve  $\rm B$  is shown in red in the above graph. The noise power density for system  $\rm A$  is drawn in blue.

For the system  $\rm B$,  the worst-case error probability

$$p_{\rm U} = {\rm Q} \left( \sqrt{\rho_{\rm U}} \right) \hspace{0.2cm}{\rm with} \hspace{0.2cm} \rho_{\rm U} = \frac{[\ddot{o}(T_{\rm D})/2]^2}{ \sigma_d^2}$$

was determined. The measurement resulted in  $p_{\rm U} = 4 \cdot 10^{\rm -8}$, which corresponds to the signal-to-noise ratio  $10 \cdot {\rm lg} \, \rho_{\rm U} = 14.8 \, {\rm dB}$. 




Notes:



Questions

1

What (normalized) noise rms value occurs in system  $\rm B$? 

$\sigma_d/s_0 \ = \ $

2

What noise rms value occurs for system  $\rm A$  when it leads to exactly the same (worst-case) error probability as system  $\rm B$? 

$\sigma_d/s_0 \ = \ $

3

By what attenuation factor  $\alpha$  is system  $\rm A$  equivalent to system  $\rm B$  in terms of (worst-case) error probability?

$20 \cdot {\rm lg} \ \alpha \ = \ $

${\ \rm dB}$

4

What is the noise power density referenced to  $N_0/2$   $($at  $f = 0)$  before the decision for system  $\rm A$  and system  $\rm B$?

$\text{System A:}\hspace{0.4cm} {\it \Phi}_{d \rm N} (f = 0)/(N_0/2) \ = \ $

$\ \cdot 10^6$
$\text{System B:}\hspace{0.42cm} {\it \Phi}_{d \rm N} (f = 0)/(N_0/2) \ = \ $

$\ \cdot 10^0$

5

For the rest of the exercise, we will only consider system  $\rm B$. At what frequency  $f_{\rm max}$  does  ${\it \Phi}_{d \rm N}(f)$  have its maximum?

$f_{\rm max} \cdot T\ = \ $

6

By what factor is the noise power density at frequency  $f_{\rm max}$  greater than at  $f = 0$?

${\it \Phi}_{d \rm N}(f_{\rm max})/{\it \Phi}_{d \rm N}(0)\ = \ $

$\ \cdot 10^6$


Solution

(1)  From $10 \cdot {\rm lg} \, \rho_{\rm U} = 14.8 \, {\rm dB}$ follows $\rho_{\rm U} = 10^{\rm 1.48} ≈ 30.2$ and continue with the given equation:

$$\sqrt{\rho_{\rm U}} = \frac{\ddot{o}(T_{\rm D})/2}{ \sigma_d}\hspace{0.3cm}\Rightarrow \hspace{0.3cm} \sigma_d = \frac{0.478 \cdot s_0/2}{ \sqrt{30.2}} \hspace{0.15cm}\underline { \approx 0.044 \cdot s_0 }\hspace{0.05cm}.$$


(2)  With the same error probability $p_{\rm U}$ (and thus the same $\rho_{\rm U}$), $\sigma_d$ must have exactly the same value as calculated in subtask (1), since the eye opening also remains the same   ⇒   $\sigma_d/s_0 \underline{= 0.044}.$


(3)  According to the specification section:

$$\alpha^2 = \frac{N_0 \cdot f_{\rm G}}{\sqrt{2} \cdot \sigma_d^2} = \frac{10^{-9} \cdot s_0^2 \cdot T \cdot f_{\rm G}}{\sqrt{2} \cdot \sigma_d^2} = 10^{-9} \cdot \frac{ f_{\rm G} \cdot T}{\sqrt{2} \cdot (\sigma_d/s_0)^2}\hspace{0.3cm} \Rightarrow \hspace{0.3cm} \alpha^2 = 10^{-9} \cdot \frac{ 0.35}{\sqrt{2} \cdot 0.044^2} \approx 1.28 \cdot 10^{-7} \hspace{0.05cm}.$$

Expressed in ${\rm dB}$, one thus obtains

$$20 \cdot {\rm lg}\hspace{0.1cm}\alpha = 10 \cdot {\rm lg}\hspace{0.1cm}\alpha^2 = -70\,{\rm dB}\hspace{0.1cm}+\hspace{0.1cm}10 \cdot {\rm lg}\hspace{0.1cm}1.28\hspace{0.15cm}\underline { = -68.9\,{\rm dB}} \hspace{0.05cm}.$$


(4)  For system  $\rm B$,  because $H_{\rm E}(f = 0) = 1$, the normalized value is equal to $1$, that means, it is ${\it \Phi}_{d \rm N}(f = 0) = N_0/2$.

In contrast, for system  $\rm A$,  this value is larger by $1/\alpha^2$ due to the components of the frequency-independent cable attenuation $\alpha$:

$${\rm System}\hspace{0.15cm}{\rm A:}\hspace{0.1cm}\frac{{\it \Phi}_{d{\rm N}}(f = 0)}{N_0/2} = \frac{1}{\alpha^2} \hspace{0.15cm}\underline {\approx 7.8 \cdot 10^{6}} \hspace{0.05cm}, \hspace{1.05cm}{\rm System\hspace{0.15cm}B}: \frac{{\it \Phi}_{d \rm N}(f = 0)}{N_0/2} \, \underline {= 1}.$$


(5)  ${\it \Phi}_{d \rm N}(f)$ is maximal if the exponent

$$18.4 \cdot \sqrt{2 f T} - 2\pi \cdot \frac{(f \cdot T)^2}{0.49}$$

has the maximum value. Thus, with $x = f \cdot T$, the optimization function is:

$$y(x) = 26.022 \cdot \sqrt{x} - 12.823 \cdot x^2 \approx 26 \cdot \sqrt{x} - 13 \cdot x^2 \hspace{0.3cm} \Rightarrow \hspace{0.3cm} \frac{{\rm d}y}{{\rm d}x} = \frac{26} {2\cdot \sqrt{x}} - 13 \cdot 2 \cdot x = 0$$
$$\Rightarrow \hspace{0.3cm} \frac{1} { \sqrt{x}} = 2 \cdot x \hspace{0.3cm}\Rightarrow \hspace{0.3cm}\frac{1} { x} = 4 \cdot x^2 \hspace{0.3cm}\Rightarrow \hspace{0.3cm} x^3 = 0.25 \hspace{0.3cm}\Rightarrow \hspace{0.3cm} x \approx 0.63 \hspace{0.05cm}.$$

This gives $f_{\rm max} \cdot T\hspace{0.15cm}\underline {\approx 0.63}$.


(6)  With $x_{\rm max} = 0.63$ we get the function value

$$y(x_{\rm max}) \approx 26 \cdot \sqrt{0.63} - 13 \cdot 0.63^2 \hspace{0.15cm}\underline {\approx 15.477}.$$
Noise component $d_{\rm N}(t)$

It follows:

  • The noise power density at the (normalized) frequency $f \cdot T \approx 0.63$ is larger than at the frequency $e^{\rm 15.5} \underline{\approx 5.4 \cdot 10^6}$ by a factor of $f = 0$.
  • Thus, periodic components with period $T_0 \approx 1.6 \cdot T$ predominate in the noise component $d_{\rm N}(t)$.
  • The graph shows a simulation and confirms this result.